2010 AMC 12A Problems/Problem 5

Revision as of 21:17, 10 February 2010 by Stargroup (talk | contribs) (Created page with '== Problem 5 == Halfway through a 100-shot archery tournament, Chelsea leads by 50 points. For each shot a bullseye scores 10 points, with other possible scores being 8, 4, 2, an…')
(diff) ← Older revision | Latest revision (diff) | Newer revision → (diff)

Problem 5

Halfway through a 100-shot archery tournament, Chelsea leads by 50 points. For each shot a bullseye scores 10 points, with other possible scores being 8, 4, 2, and 0 points. Chelsea always scores at least 4 points on each shot. If Chelsea's next $n$ shots are bullseyes she will be guaranteed victory. What is the minimum value for $n$?

$\textbf{(A)}\ 38 \qquad \textbf{(B)}\ 40 \qquad \textbf{(C)}\ 42 \qquad \textbf{(D)}\ 44 \qquad \textbf{(E)}\ 46$

Solution

Let $k$ be the number of points Chelsea currently has. In order to guarantee victory, we must consider the possibility that the opponent scores the maximum amount of points by getting only bullseyes.


$k + 10n + 4(50-n) > (k-50) + 50\cdot{10}$

$6n > 250$


The lowest integer value that satisfies the inequality is $\boxed{42\ \textbf{(C)}}$.